Ensino SuperiorIntegração por Partes Tópico resolvido

Poste aqui problemas sobre assuntos estudados no Ensino Superior (exceto os cobrados em concursos públicos e escolas militares).

Moderador: [ Moderadores TTB ]

Avatar do usuário
Autor do Tópico
Aska
iniciante
Mensagens: 3
Registrado em: Sáb 10 Out, 2009 17:18
Última visita: 29-10-09
Out 2009 29 20:01

Integração por Partes

Mensagem não lida por Aska »

Boa noite,


Tentei resolver a integral:

[tex3]\int \frac{ 2x + 2 }{ (x^{2}+1)(x-1)^{3} } dx[/tex3]


por partes (separando o denominador), mas não consigo chegar no resultado do livro! A reposta que eu encontrei foi:
Resposta

[tex3]\frac{1}{4} ln|x^{2}+1| - \frac{1}{2} ln |x-1| + C[/tex3]

Sendo C = C1 + 2
Sendo que a resposta correta:
Resposta

[tex3]\frac{1}{(x-1)^{2}} + \frac{1}{(x-1)} + tg^{-1} + C[/tex3]

Vlw!

Última edição: Aska (Qui 29 Out, 2009 20:01). Total de 1 vez.



Avatar do usuário
Cardoso1979
6 - Doutor
Mensagens: 4008
Registrado em: Sex 05 Jan, 2018 19:45
Última visita: 04-04-23
Localização: Teresina- PI
Mar 2019 24 12:36

Re: Integração por Partes

Mensagem não lida por Cardoso1979 »

Observe

Solução:

[tex3]\int \frac{ 2x + 2 }{ (x^{2}+1)(x-1)^{3} } dx[/tex3]

Usando frações parciais, temos que:

[tex3]\frac{2x+2}{(x^2+1)(x-1)^3}=\frac{Ax+B}{x^2+1}+\frac{C}{x-1}+\frac{D}{(x-1)^2}+\frac{E}{(x-1)^3}[/tex3]

Obs. O mmc de x² + 1 , x - 1 , ( x - 1 )² e ( x - 1 )³ é ( x² + 1 ).( x - 1 )³.

Daí;


[tex3]\frac{2x+2}{(x^2+1)(x-1)^3}=\frac{(Ax+B)(x-1)^3+C(x^2+1)(x-1)^2+D(x^2+1)(x-1)+E(x^2+1)}{(x^2+1)(x-1)^3}[/tex3]

[tex3]\frac{2x+2}{(x^2+1)(x-1)^3}=\frac{(Ax+B)(x^3-3x^2+3x-1)+C(x^2+1)(x^2-2x+1)+
D(x^3-x^2+x-1)+E(x^2+1)}{(x^2+1)(x-1)^3}[/tex3]

[tex3]\frac{2x+2}{(x^2+1)(x-1)^3}=\frac{Ax^4-3Ax^3+3Ax^2-Ax+Bx^3-3Bx^2+3Bx-B+Cx^4-2Cx^3+2Cx^2-2Cx+C+Dx^3-Dx^2+Dx-D+Ex^2+E}{(x^2+1)(x-1)^3}[/tex3]

[tex3]\frac{0.x^4+0.x^3+0.x^2+2x+2}{(x^2+1)(x-1)^3}=\frac{(A+C).x^4+(-3A+B-2C+D).x^3+(3A-3B+2C-D+E).x^2+(-A+3B-2C+D).x+(-B+C-D+E)}{(x^2+1)(x-1)^3}[/tex3]

Comparando os termos, resulta no seguinte sistema;

[tex3]\begin{cases}
A+C=0 →A=-C \\
-3A+B-2C+D=0 \\
3A-3B+2C-D+E=0 \\
-A+3B-2C+D=2 \\
-B+C-D+E=2
\end{cases}[/tex3]

Substituindo A = - C, nas outras três equações, resulta em um novo sistema, ou seja ;

[tex3]\begin{cases}
B+C+D=0 \\
3B-C+D=2 \\
-3B-C-D+E=0 \\
-B+C-D+E=2
\end{cases}[/tex3]

Escalonando, resulta;

[tex3]\begin{cases}
B+C+D=0 \ ( I )\\
4B+2D=2 \ (II) \\
-2B+E=0 →E=2B \ (III) \\
2C+E=2 →E=2-2C\ (IV)
\end{cases}[/tex3]

De ( I I I ) e ( IV ), vem;

E = E

2B = 2 - 2C → B = 1 - C ( V )

Substituindo ( V ) em ( I I ), temos que;

4B + 2D = 2 → 2B + D = 1 → 2.( 1 - C ) + D = 1 → 2 - 2C + D = 1 → D = 2C - 1 ( VI ).

Substituindo ( V ) e ( VI ) em ( I ), fica;

B + C + D = 0 → 1 - C + C + 2C - 1 = 0 → C = 0.

Logo,

A = 0 , B = 1 , D = - 1 e E = 2


Assim,

[tex3]\int\limits_{}^{}\frac{2x+2}{(x^2+1)(x-1)^3}dx=\int\limits_{}^{}\frac{Ax+B}{x^2+1}dx+\int\limits_{}^{}\frac{C}{x-1}dx+\int\limits_{}^{}\frac{D}{(x-1)^2}dx+\int\limits_{}^{}\frac{E}{(x-1)^3}dx[/tex3]

Substituindo os valores encontrados de A , B , C , D e E nas integrais acima, fica;

[tex3]\int\limits_{}^{}\frac{2x+2}{(x^2+1)(x-1)^3}dx=\int\limits_{}^{}\frac{A.0+1}{x^2+1}dx+\int\limits_{}^{}\frac{0}{x-1}dx+\int\limits_{}^{}\frac{-1}{(x-1)^2}dx+\int\limits_{}^{}\frac{2}{(x-1)^3}dx[/tex3]

[tex3]\int\limits_{}^{}\frac{2x+2}{(x^2+1)(x-1)^3}dx=\int\limits_{}^{}\frac{1}{x^2+1}dx-\int\limits_{}^{}\frac{1}{(x-1)^2}dx+2.\int\limits_{}^{}\frac{1}{(x-1)^3}dx[/tex3]


A integral [tex3]\int\limits_{}^{}\frac{1}{x^2+1}dx[/tex3] é imediata e é igual a arctg(x) + c.


Já as integrais [tex3]-\int\limits_{}^{}\frac{1}{(x-1)^2}dx[/tex3] e [tex3]2.\int\limits_{}^{}\frac{1}{(x-1)^3}dx[/tex3] são resolvidas através da substituição u = x - 1. Ou seja;

Fazendo u = x - 1 → du = dx , resolvendo a primeira, temos que;

[tex3]-\int\limits_{}^{}\frac{1}{u^2}du=-\int\limits_{}^{}u^{-2}du=-\frac{u^{-2+1}}{-2+1}=-\frac{u^{-1}}{-1}=\frac{1}{u}[/tex3]

Como u = x - 1, logo;

[tex3]-\int\limits_{}^{}\frac{1}{(x-1)^2}dx=\frac{1}{x-1}+k[/tex3]



Resolvendo a segunda integral, vem;

[tex3]2.\int\limits_{}^{}\frac{1}{u^3}du=2.\int\limits_{}^{}u^{-3}du=2.\frac{u^{-3+1}}{-3+1}=\cancel{2}.\frac{u^{-2}}{-\cancel{2}}=-\frac{1}{u^2}[/tex3]

Como u = x - 1 , logo;

[tex3]2.\int\limits_{}^{}\frac{1}{(x-1)^3}dx=-\frac{1}{(x-1)^2}+K[/tex3]

Portanto,

[tex3]\int \frac{ 2x + 2 }{ (x^{2}+1)(x-1)^{3} } dx=arctg(x)+\frac{1}{x-1}-\frac{1}{(x-1)^2}+C[/tex3]

Ou

[tex3]\int \frac{ 2x + 2 }{ (x^{2}+1)(x-1)^{3} } dx=-\frac{1}{(x-1)^2}+\frac{1}{x-1}+arctg(x)+C[/tex3]

Ou ainda;

[tex3]\int \frac{ 2x + 2 }{ (x^{2}+1)(x-1)^{3} } dx=\frac{x+(x-1)^2arctg(x)-2}{(x-1)^2}+C[/tex3]


Nota

Você se equivocou tanto na sua resolução como no gabarito do livro postado por você.


Bons estudos!




Responder
  • Tópicos Semelhantes
    Respostas
    Exibições
    Última msg

Voltar para “Ensino Superior”